Loading [MathJax]/extensions/TeX/boldsymbol.js

5 votos

La necesidad del campo B

Es bastante fácil, utilizando la relatividad especial básica, llegar a la conclusión de que el efecto de la fuerza magnética sobre las cargas cercanas de los cables que transportan corrientes sobre las cargas cercanas sólo se debe a la contracción de la longitud en ciertos marcos inerciales desde el punto de vista de las cargas, lo que da lugar a un cambio percibido en la densidad de carga. Sin embargo, estructuramos las leyes (clásicas del campo) del electromagnetismo en las leyes de Maxwell utilizando un campo B.

Mi pregunta es, ¿es posible formular las ecuaciones de Maxwell sólo en términos de campos E transformados en Lorentz? Si no es así, ¿por qué no?

2 votos

Este tema ha sido objeto de un gran debate aquí: physics.stackexchange.com/questions/3618/

0 votos

0 votos

Muchas gracias por señalarlas, disculpas por la repetición

5voto

Alexey Lebedev Puntos 4778

No. \boldsymbol{B} es una entidad independiente.

Referencia Jackson, Classical Electrodynamics, Sección 12.2.

Jackson argumenta que el tensor antisimétrico F formulación de EM que todos conocemos y amamos: m \frac{\mathrm d^2x^{\alpha}}{\mathrm d\tau^2} = q F^{\alpha\beta} \frac{\mathrm dx_{\beta}}{\mathrm d\tau} no es la única generalización covariante posible de la ley de fuerza del marco de reposo: ma=qE
De hecho, construye un contraejemplo basado en un potencial escalar de Lorentz \phi . De forma no relativista, este potencial da una fuerza de Coulomb y una fuerza magnética, tal y como vemos, pero no hay un \boldsymbol{B} -campo: en cualquier marco, la fuerza está dada en términos del gradiente 4 del potencial escalar, \partial_\mu \phi y no los 6 componentes de F . Si este escalar de Lorentz fuera la forma en que funciona el mundo, la respuesta a tu pregunta sería sí. Pero no es así.

2voto

heathrow Puntos 25

Sí--- el electromagnetismo se desarrolla desde este punto de vista, como adecuado para un curso de pregrado, en Purcell. La ecuación del movimiento es

ma = qE

donde E y a están ambos en el marco de reposo. La forma covariante de esta ecuación es la ecuación de movimiento habitual:

m {\mathrm d^2x_{\nu}\over \mathrm d\tau^2} = q F_{\mu\nu} {\mathrm dx^\mu\over \mathrm d\tau}

Como se puede ver especializando al marco de reposo, así la ley de Newton en el marco de reposo, sólo usando E, es efectivamente covariantemente equivalente a ambos E y B en el marco habitual.

0 votos

La covarianza de la primera ecuación sólo significa ma' = qE' para u'= 0 . La segunda ecuación implica la primera, y no al revés.

0 votos

@LarryHarson: Bajo el supuesto de la covarianza, el caso especial a bajas velocidades implica lo general: una vez que conoces el comportamiento a bajas velocidades, encuentras el comportamiento a altas velocidades mediante el impulso. ¿Cómo encontrar rápidamente el comportamiento covariante correcto? Se encuentra una expresión convariante que se reduce al límite no relativista correcto, en este caso determinado de forma única, a partir de la forma tensorial de F, que se deduce de la invariancia de la carga bajo impulsos. Así es como se deriva la segunda ecuación a partir del caso especial de la primera, y es como lo hizo Einstein.

0 votos

Si ya sabes cómo E se supone que se transforma, entonces sí, la covarianza significa que la primera ecuación implica la segunda. Pero esto es demasiado fácil. ¿Te refieres al método de Einstein en su artículo de 1916 sobre la RG?

2voto

Oeufcoque Penteano Puntos 331

Creo que la clave aquí es su exigencia de una formulación "en términos de Transformación de Lorentz Campos electrónicos".

El campo E es un campo de 3 vectores. Un campo covariante de Lorentz debe ser un campo de 4 vectores o 4 tensores (un campo de Lorentz invariante debe ser un campo escalar de Lorentz).

De hecho, el campo E es, dentro de SR, un componente de un rango 2 campo electromagnético de 4 tensores mientras que los potenciales escalares y vectoriales son componentes de a potencial electromagnético de 4 vectores .

Dado que el campo E es sólo una parte de un tensor covariante de Lorentz, la noción de "campo E transformado de Lorentz" necesita una mayor aclaración.

1voto

gotgenes Puntos 288

Si se mira el formalismo covariante de la electrodinámica clásica , puede ver que no tiene que mencionar ni el \bar{E} o \bar{B} campo. Puedes hacer todos tus cálculos con el potencial de cuatro A_\mu y el tensor electromagnético F_{\mu\nu} .

i-Ciencias.com

I-Ciencias es una comunidad de estudiantes y amantes de la ciencia en la que puedes resolver tus problemas y dudas.
Puedes consultar las preguntas de otros usuarios, hacer tus propias preguntas o resolver las de los demás.

Powered by:

X